Tải bản đầy đủ (.pdf) (20 trang)

SH giang QV cap cua so nguyen can nguyen thuy va ung dung

Bạn đang xem bản rút gọn của tài liệu. Xem và tải ngay bản đầy đủ của tài liệu tại đây (785.29 KB, 20 trang )

Báo cáo chuyên đề số học
Cấp của số nguyên - căn nguyên thủy và ứng dụng.
Tác giả : Quách Văn Giang
Trường THPT chuyên Hà Nội Amsterdams
Cấp của số nguyên và căn nguyên thủy của số nguyên có nhiều ứng dụng trong
việc giải một số lớp bài toán số học. Trong bài viết này tôi muốn cùng trao đổi
với các bạn đồng nghiệp về một vài ứng dụng nhỏ của cấp số nguyên thủy để
giải bài toán số học thuộc các dạng chứng minh sự tồn tại của một lớp các số
nguyên có tính chất số học cho trước và bài toán tìm kiếm các số tự nhiên khi
biết tính chất số học của chúng. Thông qua bài viết này tôi cũng mong muốn
được sự chia sẻ, trao đổi của các bạn đồng nghiệp trong các trường chuyên.
I) Những kiến thức cơ bản về cấp và căn nguyên thủy
1) Những khái niệm cơ bản
Định nghĩa : + Cho a,n là các số nguyên dương nguyên tố cùng nhau. Số
nguyên dương k nhỏ nhất thỏa mãn diều kiện a k  1(mod n) gọi là cấp của a theo
modulo n. Ký hiệu là k  ord n (a)
+ a được gọi là căn nguyên thủy modulo n nếu ordn (a)   (n)
2) Một số tính chất của cấp
a) Tính chất 1
Nếu (a,n) = 1, h  ord n (a) k  N* mà a k  1(mod n)  k h
Chứng minh : Giả sử k = th+r
0  r  h  1, a h  1(mod n)  a k  1(mod n)  a r  1(mod n)  r  0  h / k

b) Tính chất 2
Nếu ordn (a)  h, ordn (h)  l, (h, l)  1thì ordn (ah)  hl
Chứng minh : Đặt t  ordn (ab)
Ta có (ab)hl  a hl .bhl  1(mod n)  t / hl
(ab)th  a th .bth  bth (mod n)
 l / th  l / t
Ta có  t
th


(ab)  1(mod n)  (ab)  1(mod n)


Tương tự 1  (ab)th  ath .bth  btl (mod n)  h / tl  h / t
t / hl , h / t , l / t
 t  hl (dpcm)
(h, l )  1

Vậy ta có 

c) Tính chất 3
Cho n1 , n2 ,..., nk là các số nguyên tố cùng nhau đôi một. Đặt n  n1 , n2 ,..., nk . Giả sử
rằng hi  ord n (a) khi đó ta có ordn (a)  [ h1 , h2 ,..., hk ]
i

Chứng minh : Đặt
h  ordn (a)  ah  1(mod n)  a h  1(mod ni )i  1,k  hi / hi  1,k  h / a

Là bội số chung của h1 , h2 ,..., hk  al  1(mod ni )i  1, k do n1, n2 ,..., nk đôi một
nguyên tố cùng nhau  a l  1(mod n )  h / l vậy l  [h1 , h2 ,..., hk ] (đpcm)
d) Tính chất 4
(a, n)  1  ord n (a) /  (n) ( n ) .

Áp dụng định lý Euler do (a, n)  1  a ( n )  1(mod n)  ord n (a) /  (n)
e) Tính chất 5
(a, n)  1, n  0 a i  q i (mod n)  i  j (mod ord n a)

i  k h  r
Đặt  i i  a r  a r (mod n) GS ri  rj
i  k j h  rj

r  rj

 a ri (a i

i

j

r  rj

 1)  0(mod n)  a i

 1(mod n)  ri  rj  0  i  j (mod h) đpcm

f) Tính chất 6
Giả sử ord m (a)  t , u  N * khi đó ord m au 

t
u

Chứng minh : Đặt ordm (au )  h  a hu  1(mod m)
t / hu
t / hu
t

 t  hu  h 
u
hu / tu
h / t


Mà t  ordm a  atu  1(mod m)  

3) Định lý về cấp với modun nguyên tố.
a) Định lý 1


Cho a là số nguyên dương lẻ n  2S (s  N * ) Gọi h = ordn (a ) nếu
a  1  2u.b, a 2  1  2c với c  b  1(mod 2) và 1  u  v khi đó h = 1
nếu u  S , h  2t , t  max 1; S  1  v nếu u < v
- Chứng minh : Theo định lý Euler a ( n)  1(mod n)  h /  (n)   (2s )  2S 1
Nếu u  s  2S 1 / 2u.b  a  1(mod n )  h /1  h  1
t 1

do h / 2s1  h  2t , t  s  1  t  1, h  2  a2  1  (a 2  1)(a 2  1)(a 4  1)...(a 2  1)
t

2 t 1

 c.2v (a2  1)...(a 2

 1)

a 2 n  1  0(mod 2)
a h  1  0(mod 2v t 1 )
 h
Do  2 n
v 1
a  1  b(mod 4), b  0 a  1  0(mod 2 )

Nếu u  s  v  2s / a 2  1  c.2v  h / 2  h  2

a h  1 2v t 1

Nếu s  v  

h
s
a  1 2

 v  t 1  s  t  s  1 v

Vậy h  2s 1v (đpcm)
b) Định lý 2
Cho p  P , p lẻ, (a,p) = 1, n  ps (s  N * ), r  ord p (a) và a r  1  pu q  1 đặt
(u  s)
r
h  ordn (a) khi đó ta có h   s u
us
 r. p

Chứng minh :
- Ta chứng minh bổ đề (a, p)  1, a  1  p.q thì a p 1  pn .A (A, p)  1
n

n  0  a p 1  a 1  p .q ( p, q)  1  mệnh đề đúng n = 0
o

Giả sử a p 1  a 1  pn .A, ( A, p)  1
n

đặt b  a p  (b, p)  1  b p 1  (b 1)(b p1  b p2  ...  p  1)  (b 1).B

n

b  1  a p  1  p n  . A, ( A, p)  1

 B  p  0(mod p)

 B  B1 p ( B1 , p)  1
b  1(mod p)  
2
 B  0(mod p )

n

 b p  1  p n 1. AB1 ( AB1 , p)  1  a p

- Chứng minh định lý :

n

1

 1  p n 1. A0 , (A0 , p)  1 (đpcm)


a) Nếu r =1 hay a  1(mod p) theo giả thiết a  1  pu .q, ( p, q)  1
Nếu u  s  a  1(mod ps )  a  1(mod n)  h  1
Nếu u  s  h  pt .q, ( p, q)  1
Nếu q  1  ah 1  a p .q 1 đặt d  a p  ah 1(d q 1)  (d 1)(dq1  d q2  ...  d  1)
t


t

Do d  1(mod p)  dq1  d q2  ...  d  1  q  0(mod p)  (ah 1)  (a p 1).b (b, p)  1
t

Vậy a p  1(mod n)  h/ p t vô lý vậy q  1  h  p t
t

Theo bổ đề a h  1  pt 4 A, ( A, p)  1  t  a  s  t  s  a vậy t nhỏ nhất là
t = s - u h  p s u
b) r bất kì do a h  1(mod n)  a h  1(mod p)  h  r
đặt b  a r  a h  bl
1 (u  s)

dễ thấy ord p (b)  1  l  ord n (b)  

p

s u

r (u  s)
 ord na  rl   s r
(u  s )
(u  s )
rp

II) Ứng dụng cấp căn nguyên thủy giải một số bài toán số học
1) Sử dụng cấp của số giải bài tập chứng minh sự tồn tại của số nguyên có
tính chất số học xác định.
Bài 1 : Cho a, b  N * (a, b)  1 đặt An  a 2  b2 , n  N * .Chứng minh rằng với mọi

ước lẻ của An có dạng 2n1.k  1, k  N *
n

n

Lời giải
Gọi p là ước nguyên tố lẻ bất kỳ của An
Ta có An  0(mod p)  a 2  b2 (mod p) (1)
n

n

 ( a , b )  1 ( a , p )  1

 b'  0,1, 2,..., p  1 sao cho b.b '  1(mod p)
pP
(b, p)  1

Do 

Từ (1)  (b2 )2  (a 2 )2 (mod p)  a 2  b2 (mod p)   ab '
n

n

n1

n1

Gọi h  ord p ( ab ') theo định lý Fecma (ab ') p1  1(mod p) (3)

h / 2n1
Từ (2) và (3)  
 h  2 ,   0,1,..., n, n  1
h / p  1

2n1

 1(mod p) (2)


n



Nếu  {0,1, 2,..., n}  (ab ')2  [(ab ')2 ]2  1(mod p )  (ab ')2  (bb ')2 (mod p )
n

n

n

mà (bb ')2  1(mod p)  a 2  b2 (mod p)  2.a2  0(mod p)  p / 2 vô lý.
n

n

n

n


Vậy   n  1  2n1 / p 1  p  k.2n1 1 ( k  N * )
Bài 2: Chứng minh rằng tồn tại vô số số nguyên tố p sao cho q / 2 p 1 và
p / 2q 1  1

Lời giải.
Xét dãy Fn  22  1, n  1 Xảy ra 3 trường hợp.
n

n1

a) Fn  P. đặt Fn  p  P giả sử q / Fn1  22  1
2n

n1

Ta có 2 p 1  1  22  1  (22  1)(22  1)(22  1)...(22  1) (1) vì 2n  n  1 nên trong
1

1

2

dạng phân tích ở vế phải của (1) có chứa 22  1 vậy 2 p 1  0(mod Fn1)  0(mod q)
n1

n1

Vậy 2 p1  1(mod q) theo giả thiết 22  0(mod q )  Fn  (22 )2  42  12
n


n

n

(4,1) = 1 theo kết quả bài 1 :
n1

n1

q  t.2n1  1  2q1 1  2t .2 1 22 1  (22 1)(22  1)  2q1  1(mod p)
n

n

 2 p 1  1(mod q)
Vậy  q 1
 2

 1(mod p)

b) Fn là hợp số và Fn có 2 ước nguyên tố p,q khác nhau.
Giả sử p, q / Fn1  22  1
n

n 1
p 1
x.2

 1 22  1  (22  1)(22  1)  2 p 1  1(mod q)
 p  x.2  1 

2  1  2


n
n 1
q 1
y .2n1

 1  0(mod 22  1)  0(mod p)  2q 1  1(mod p)
q  y.2  1 
2  1  2
n1

n1

n

n

c) Nếu
Fn  22  1  p k , k  N * , k  2  22  p k  1  ( p  1)(1  p  ...  p k 1 )
n

n

 1  p  ...  pk 1  k (mod 2)  k ( p  1)  0(mod 22 )
n

Do p > 2  k  0(mod 2)  1  22  x2  22  ( x  1)( x 1)
n


u

u  1
x 1  2
v
u

2

2

2

 vô lý


v
v

2
x

1

2





Bài 3 : Cho n  N * , n  1.

n


a) p là ước nguyên tố của a 2  1 . Chứng minh rằng nếu a là số nguyên
dương thỏa mãn  x : a  x 2 (mod p) thì p  1(mod 2n 2 )
n

b) p là ước nguyên tố của 22  1 . Chứng minh rằng p  1(mod 2n2 )
n

Lời giải
a) Vì  x : a  x 2 (mod p)  a

p 1
2

 1(mod p) (tiêu chuẩn Euler)

Theo giả thiết a  1  0(mod p)  a
2n

2n1

 p21

 1(mod p) (1)
 1  0(mod p)  a
n1

2

 1(mod p)
a

Gọi h  ord p (a)  ah  1(mod p) (2)
n
 h / 2 n 1
 2  1(mod p )

n 1
n a
Từ (1) và (2)   p  1 nếu h  2  h / 2  n
 2  0(mod p ) Vô lý
2
a


1(mod
p
)
h /



2

Vậy h  2n 1 

p 1

 0(mod 2 n 1 )  p  1(mod 2 n  2 )
2

b) Theo kết quả bài 1 thì ta  p  1(mod 2n1 )
p 1
2
(8q  1) 2  1
8
 (1)8 q  2  1
Do n  1  p  1(mod 8)  ( )  (1)  (1)
p
8
2

p 1

 p21
p 1
2
2  1(mod p) h  ord p (2) /
2

Vậy  1  2  1(mod p)   n1
2  h  2 n 1
p
n 1
22  1(mod p)

h / 2



p 1
 0(mod 2n 1 )  p  1(mod 2n  2 )
2

Bài 4 : (China tst 2005) Chứng minh rằng ước nguyên tố lớn nhất của 22  1
không bé hơn (n  1).2n2  1 .
n

Lời giải:
Giả sử 22  1  P1k .P2k ...Prk với P1  P2  ...  Pr
n

1

2

r

Theo chứng minh trên ta được:
Pi  1(mod n  2)  qi : Pi  qi 2n2  1, i  1, r

Do P1  P2  ...  Pr  q1  q2  ...  qr do n  2  2n  2n  4


 22  1  P1k1 .P2k2 ...Prkr  1(mod 22 n  4 )
n

 (q1 2n  2  1) k1 (q 2 2n  2  1) k2 ...(q r 2n  2  1) kr  1(mod 22 n  4 )
r


 1  2n  2  ki qi  1(mod 22 n  4 )
i 1

r

 2n  2  ki qi  0(mod 22 n  4 )
i 1

r

  ki qi  0(mod 2n  2 )
i 1

r

r

i 1

i 1

 2n  2   ki qi  qr  ki

(1)

Mặt khác,
r

22  1   (q i 2n  2  1) ki  (1  2n  2 ) k1  k2 ... kn  2

n

( n  2)

 ki  1

i 1

 22  2( n  2) k1  k2 ... kn  k1  k2  ...  kn 
n

2n
n2

r
2n
 qr
 qr  qi  2n  2  qr  4(n  2)
n2
i 1

 Pr  qr .2n  2  1  (n  2)2n  4  Pr  (n  2)2n  4  1

Bài 5: Hàn Quốc TST 2003. Cho p  , đặt f P ( x)  x P 1  x P 2  ...  x  1
1) Chứng minh rằng p/m thì mọi ước nguyên tố của f P (m) đều nguyên tố cùng
nhau với m(m - 1).
2) Chứng minh rằng tồn tại vô hạn số nguyên dương n để pn+1 là số nguyên
tố.
Lời giải:
1) gọi q là ước nguyên tố của f P (m)  f P (m)  0(mod q)

Do m  0(modp)  f P (m) 1(mod p)  (p,q)  1  (q, m)  1
Đặt d  (q, m  1)
Nếu d  1  q /  m 1  m  1(mod q)
 0  f p (m)(mod q)  p(modq)  p  0(modp)  m  0(modq)

(q, m)  1
  q, m( m 1)   1
(q, m 1)  1

Điều này vô lý, vậy d  1  

2) Ta chứng minh tồn tại vô hạn số nguyên tố q sao cho p/q-1


Thật vậy, gọi q là ước nguyên tố bất kỳ của f P (m)
Do f P (m) / (m 1) f P (m)  m p  1  q / m p  1
Đặt h  ordq (m)  mh  1(mod q)  h / p  h 1, p
Nếu h  1 m  1(mod q ) mâu thuẫn với câu a  h  p
Do m p  1(mod q)  (m, q)  1  nq1  1(mod q)  p / (q  1)
Từ chứng minh trên ta xây dựng dãy mk k 1 :  f P (mk ) có vô hạn ước nguyên tố
Xét dãy m1  p, mk  pf P (m1 ). f P (m2 )... f P (mk 1 )
 f P (m1 ). f P (m2 )... f P (mk 1 ) / m k
 f P (mk )  f (0)  m kp 1  m kp  2  ...  m k  0(mod mk )
 f P (m1 ). f P (m2 )... f P (mk 1 ) / f P (mk )  f (0)  f P (mk )  1
  f P (mk ), f P (mi )   1 i  1, k  1

Vậy  f P (mk )k 1 có vô hạn ước nguyên tố khác nhau (đpcm)
Bài 6: Cho p  5 , p  P
1) Chứng minh q  P, q  p sao cho qn /  ( p  1) p  1
p2

2) Giả sử ( p  1)  1   q , chứng minh rằng  ai qi    p 2 
2
i 1
i 1
n

n

ai
i

p

Lời giải:
1) Do p  5 , p  P  p  1(mod 2)  ( p  1) p  1  0(mod p)  ( p  1) p  1 là hợp số.
Ta có p p1  p p1  1  p p  tồn tại ước nguyên tố q của p p 1  1 và q  p .
n

A

ai qi



i 1
2) Đặt 
n
B  a

i


i 1

Áp dụng bất đẳng thức AM.GM
 n
n
  ai qi
ai
p 1
p  1   qi   i 1
 B
i 1










B

 A  B.B  p  1  1
p

(1)



Trường hợp 1, nếu B 

p 1
p
 A  B B p p 1  B p 2 B  p 2 
2
2

Trường hợp 2:
p 1
B p
2

 A  B B ( p  1) p  1 

p 1 B
p  1 B p 1 p  1 pB1 p( p  1) p 2
( p  1) p  1  p 2 
p 
.p 

2
2
2
2
2

Trường hợp 3: Nếu B  p, q / ( p  1) p  1  ( p  1)2 p  1(mod q )
h  p
h  2

Đặt ord q ( p  1)  h  h / q  1  
h  2 p
h / 2 p


Nếu h  2  ( p  1)2  1(mod q )  p ( p  2)  o (mod p )
 ( p  2)  0(modq) Hay p  2(modq)

 0  ( p  1) p  1 (mod q)  2(mod q)  q  2 ( vô lý) và h=2p

 2 p / (q  1)  q  1  2 p  p  pi  p
 A  p(a1  a2  ...  an )  Bp  p 2 (dpcm)

Bài 7: Cho r  P đặt f r ( x)  x r 1  x r  2  ...  x  1
Chứng minh rằng f r ( x) chia hết cho vô hạn số nguyên tố khi x nguyên tố.
Lời giải:
Trước hết ta có định lý Đirichlet về số nguyên tố cho a, b  N * ,  a, b   1
Khi trong T( a,b)  b  ka; k  N chứa vô hạn số nguyên tố
Phản chứng giả sử f r ( p) chỉ chia hết cho hữu hạn số nguyên tố q1 , q 2 ..., q k
Theo định lý Đirichlet, tồn tại l sao cho p  lq1 q 2 ...q k  1  P
f r ( p)  r  0(modqi ) với i 1, 2,..., k  r  qi

j  i , r  q j  f r ( p)  r  0(mod q j )  f r ( p)  q Sj  r S

Từ ri  qi  p  1  lq1 q 2 ...q k  0(mod q j )
 pr 1 
r
  vr  p  1  vr  p  1  vr  p  1  vr (r )  vr  p  1  1
p


1



Ta có vr  f r ( p)   vr 


 f r ( p)  r

Tuy nhiên với p đủ lớn thì f r ( p)  r , mâu thuẫn , suy ra đpcm
Bài 8: Cho p là số nguyên tố lớn hơn 3. Chứng minh rằng với mọi ước nguyên
dương của

2p 1
đều có dạng 2kp+1.
3

Lời giải:
Trước hết ta nhận thấy rằng: v3 (2 p  1)  v3 (2  1)  v3 ( p)  1
Suy ra mọi ước nguyên tố của

2p 1
đều lớn hơn 3.
3

2p 1
Gọi q là ước nguyên tố bất kì của
 2 p  1  0(mod q)
3
 22 p  1  0(mod p) hay 22 p  1(mod 2)


Giả sử h  ord p (2)  h / 2 p  h 1, 2, p, 2 p
Nếu h  1  1  0(mod q )  q / 1 vô lý.
Nếu h  2  3  0(mod q )  q / 3  q  3 vô lý.
2 p  1(mod q )

Nếu h  p  

p
2  1(mod q )

 2  0(mod q )  q/ 2 vô lý

Vậy h  2 p  q  1  0(mod 2 p)  q  1(mod 2 p)
Do đó q=2kp+1 (đpcm)

3) Sử dụng cấp, căn nguyên thủy giải bài toán tìm các số tự nhiên biết
tính chất số học của chúng
Bài 9:(China MO 2005) Tìm tất cả các cặp số nguyên tố p ,q sao cho pq là
ước của 5 p  5q
Lời giải
Ta xét hai trường hợp :
a) p, q 2,5  5 p  5  0(mod p)  5 p  5q  5q  5  (mod p)  5q1  1  0(mod p) (1)
Từ (1) và định lý nhỏ Fermat ta nhận được


52( q 1)  1(mod p) h  ord p (5) | 2(q  1)

 v2 ( p  1)  v2 (h)  v2 (q  1) (3)
 p 1

5  1(mod p)
h | p  1, q  1  0(modp)

Chứng minh tương tự ta cũng có
mâu thuẫn.

v2 (q  1)  v2 (q  1) (4) .Từ (3) và (4) suy ra

Vậy trường hợp này không có p , q thỏa mãn yêu cầu bài toán
b)Trong hai số p,q có ít nhất một số thuộc tập 2,5 .Giả sử p 2,5
i) nếu p=2  5q  5  0(mod 2q)  q=2 không thỏa mãn
5q  5  5(5q 1  5)  0(mod p )
 5q  5  0(mod 2 q)
ta có  q
5  5  0(mod 2)

30  0(mod 2q)  q/ 30  q 3,5 , thử trực tiếp thỏa mãn.

ii) p = 5, 5q  3125  0(mod 5q)
nếu q = 5 (thỏa mãn)
5q  5  0(mod 5)

nếu q  5 

5  5  0(modq)
q

 5q  5  0(mod 5q)

3130  0(mod 5q)  3130  0(modq) , q=2 và q=313 thỏa mãn.


Vậy ( p, q) (2,3),(3, 2),(2,5),(5, 2),(5,5),(5,313) ,(313,5)
Bài 10: Cho k  N * , k  2 . Tìm tất cả n  N * sao cho 2n  1 nk
Lời giải
k  N * , n  1 thảo mãn yêu cầu bài toán.

Xét n >1 . Gọi p là ước nguyên tố nhỏ nhất của n, do
2n  1  1(mod 2)  n  1(mod 2)  p  1(mod 2)  p  3

Theo giả thuyết, (2)n  1(mod nk )  (2)n  1(mod p)

(1)

Mà (-2,p)=1 nên theo định lý Fermet: (2) p1  1(mod p) (2)
h / n
h / p  1

Gọi h  ord p (2)  

 p 1  h  q
 p  1  h  q  p vô lý,
q / n

Nếu h>1 , suy ra h có ước nguyên tố q  

vậy h = 1, hay 3  0(mod p)  p  3  v3 (n)  1 (3)


Nếu k  3 , từ 2n  1 n k  v3 (2n  1)  v3 (n k )
 1  v3 (n)  k .v3 (n)  v3 (n) 


1
1

k 1 2

 v3 (n)  0 (4) . Ta thấy (3) và (4) mâu thuẫn. vậy nếu k  3 thì không tồn tại

n>1 để 2n  1 nk .
Nếu k  2  v3 (n)  1 (5) , từ (3) và (5)  v3 (n)  1 (6)
Theo chứng minh trên ta có
n  3m, m  N *  (8)m  1(mod 9m2 )  (8)m  1(mod m2 )

Nếu m  1  n  3 , thử lại 23  1  9 9  32  n  3 thỏa mãn.
Nếu m >1. Gọi q là ước nguyên tố bé nhất của m  q  3
Ta có (8)m  1(mod q) va (8)q1  1(mod q) (7)
l/ m
l/ p  1

Giả sử l  ord q (8)  

Nếu l >1 thì l có ước nguyên tố r  q  1  l  r  q vô lý , vậy l=1
 9  0(mod q)  q  3  v3 (n)  2 (8)

(6) và (8) mâu thuẫn, vậy m >1 bài toán vô nghiệm.
Kết luận k  3 n  1 là số nguyên dương duy nhất cần tìm
k  2 n 1,3

Bài 11: Tìm tất cả các cặp số nguyên tố (p,q) sao cho p.q / p q  q p  1
Lời giải

a) nếu p  q  p 2 / 2 p 2  1, do p  2  p 2 / 2 p p  p 2 /1 vô lý.
b) p  q không giảm tính tổng quát ta giả sử p  q
nếu p = 2, ta cần tìm q  P, q  3, 2q / q 2  5  q / 5  q  5
vậy ta có (2,5) là cặp thỏa mãn.
Nếu p  3 , do đó q  p  2 theo giả thuyết p q  q p  1  0(mod q)  ( p )q  1(mod q )
Do p,q nguyên tố khác nhau nên p,q nguyên tố bằng nhau.


Áp dụng định lý Fermat, ta có ( p)q 1  1(mod q) , gọi
h/ p
h  1
h  ordq ( p)  

h/ q  1  h  p

Nếu h  1  p  1  0(mod q )  p  1  q  p  2 vô lý
Nếu h  p  q  kp  1  p / p p   kp  1  1  p / 2 vô lý
q

Vậy chỉ có 2 cặp (p,q) thỏa mãn là (2,5) và (5,2)
 p / qr  1

Bài 12: (USA-MO-2003) Tìm tất cả bộ ba số p,q,r sao cho: q / r p  1
r / p q  1


Lời giải
Giả sử (p,q,r) là bộ ba số nguyên tố thảo mãn yêu cầu đề bài.
Nếu p = q  p / p r  1  p /1 vô lý
Nếu q  r  q / q p  1  q /1 vô lý

Nếu r  p  r / r q  1  r /1 vô lý
Vậy p,q,r đôi một khác nhau.
Không giảm tổng quát ta giả sử rằng p  min( p, q, r ) .
Nếu p >2 thì p lẻ, từ giả thiết ta suy ra ( p)q  1(mod r ) . Mà p,r nguyên tố cùng
nhau nên theo định lý Fermat, ta có: ( p)r 1  1(mod r ) .
h/ q
h  q

h/ r  1  h  1

Gọi h  ord r ( p)  

Nếu h = 1, thì p  1  0(mod r )  p  1  r  p  2 vô lý
Nếu h = q thì r  kq  1  q / (kp  1) p  1  q / 2 vô lý
r 2  1  0(mod q )
.
q
2  1  0(modr)

Vậy p = 2, ta có: 

 t/ q
t  q

 t/ r  1 t  1

Gọi t  ord r (2)  

Nếu t = 1, thì 3  0(mod r )  r  3  10  0(modq)  q  5
Nếu t = q thì r  kq  1  q / (kp  1) p  1  q  2 vô lý



Vậy ta có các bộ (p,q,r) là (2,5,3), (5,3,2), (3,2,5)
Bài 13 : Tìm tất cả các số nguyên dương a, sao cho với a đó thì tồn tại số
nguyên dương n lớn hơn 1 mà a n  1 n 2
Lời giải
a 2 k  1(modn)
(1)
Nếu a + 1 có ước là n, a  1(modn)   2 k 1
 1(modn)
a
n 1

Do n lẻ a n  1  (a  1) (1)i a i
i 0

n 1

Từ (1)   (1)i ai  n  0(mod n) mà a  1  0(modn)  a n  1  0(modn 2 )
i 0

Vậy nên a  N * , a  1 có ước lẻ thỏa mãn điều kiện bài toán.
Giả sử a  N * , n  1: a n  1  0(modn 2 ) ta chứng minh n lẻ.
Thật vậy, nếu n  0(mod 2)  n2  0(mod 4)  an   1(mod 4) vô lý do n chẵn, suy ra
n lẻ.Gọi p là ước nguyên tố lẻ bé nhất của n.
Từ a n  1  0(modn 2 )  a n  1  0(mod p) Do n lẻ  (a)n  1(mod p)  (a, p)  1
h/ n
h/ p  1

Theo Fermat (a) p 1  1(mod p) Gọi h  ord p (a)  


q / n
 p  1  q  p vô lý
q / p  1

Nếu h >1 suy ra h là ước nguyên tố của q  
Vậy h =1  q  1  0(mod q)

Do đó (a +1) có ước lẻ, vậy các số cần tìm là a  N * , a  1 có ước lẻ.
Bài 14: Tìm tất cả các cặp số nguyên dương (m,n) thỏa mãn:
2n  (n   (n)  1)  nm  1

Lời giải
Nếu n có nhiều hơn một ước nguyên tố. gọi p là ước nguyên tố nhỏ nhất của n,
q là một ước nguyên tố còn lại, ta có:
n   (n)  1  pq  ( p  1)(q  1)  1  p  q  2  p

Do đó p / (n   (n)  1) mặt khác, p / nm  p / 2n  1


h/ n
 h / (n, p  1)  1 , do p là ước nguyên tố nhỏ nhất của n
h/ p  1

Gọi h  ord p (2)  

 h  1  p / 21 1  1 vô lý

Vậy n  p k , p  P
Nếu k  3  n   (n)  1  p3  p 2  1  p , lập luận tương tự như trên ta suy ra mâu

thuẫn.
a) Vậy n  p 2  n   (n)  1  p 2  ( p  1)  1  p  1
nếu p  2  n   (n)  1  p2  p 1  p , lập luận tương tự, mâu thuẫn
do đó p  2  n  4  4m  16  m  2
b) Nếu n = p, thay vào (1)  2 p  p m  p  2, m  3
có hai cặp (m,n) thỏa mãn là (2,4) và (2,2) thỏa mãn ycbt.
Bài 15:Tìm tất cả các bộ số nguyên dương (a,b,n) thỏa mãn
a  b, (a, b)  1 và a n  b n n a
Lời giải
Dễ thấy (a,a,n) và (a,b,1)với a, b,n nguyên dương thỏa mãn yêu cầu bài toán
Ta chỉ xét các bộ nguyên dương (a,b,n) thỏa mãn n > 1 , a > b ,(a,b) =1
gọi p là ước nguyên tố nhỏ nhất của n, ta có a n  bn (mod p)
( a , p )  1
 a p 1  b p 1  1(mod p)
(b, p)  1

do (b,p)=1  

do đó a( n, p 1)  b( n, p 1) (mod p) .
Do p là ước nguyên tố nhỏ nhất của n nên  (n, p  1)  1  a  b(mod p)
n
n

v p (a  b )  v p (a  b)  v p (n)
Nếu p lẻ  
 v p (a  b)  (a  1)v p (n)  a  1
a

v p (n )  a.v p (n)


Do đó (a  b)  pa1
Mà a  1  pa1  a  1  a  b  pa1  a  1 , vô lý, vậy p=2.
Vậy a  b  0(mod 2)
Ta có v2 (an  bn )  v2 (a2  b2 )  v p (n) 1, v2 (nn )  a.v2 (n)


 v2 (a 2  b 2 )  v p (n)  1  a.v2 (n)
 v2 (a 2  b 2 )  (a  1).v2 (n)  1  a
a  3
 a 2  b 2  2a  
b  1

v2 (n)  1

Vậy ta cần tìm n : 

3
n / 3  1

Giả sử n có ước nguyên tố lẻ. gọi q là ước nguyên tố lẻ nhỏ nhất của n
3n  1(mod q )
 3( n , q 1)  1(mod q )
Theo giả thiết 3  1  0(mod n )   q 1
3  1(mod q )
n

3

Do q là ước nguyên tố lẻ nhỏ nhất của n,
v2 (n)  1  (n, q 1)  2  8  0(modq)  q  2 , vô lý


Vậy n = 2, do đó (3,1,2) thỏa mãn yêu cầu.
Kết luận các bộ thỏa mãn (a,b,n), (a,b,1), (3,1,2)
Bài 16: Tìm tất cả các số nguyên dương n sao cho n và 2n  1có cùng tập ước
nguyên tố.
Lời giải:
Cho p  P, p  5 , thì có nguyên tố khác 3 của 2 p  1
Ta có v3  2 p  1  1  v3 ( p)  1  2 p  1 có ước nguyên tố q khác 3
Ta có 22 p  1(mod q) , gọi h  ordq (2)  h / 2 p
Do 2 p  1(mod q)  h  2 p
Theo Fermat 2 p1  1(mod q)  2 p/ q1  q  2 p 1  p
Giả sử n là số nguyên dương thỏa mãn ycbt  n  1(mod 2)
Nếu n có ước nguyên tố khác 3. Gọi r là ước nguyên tố khác 3 lớn nhất đó,
theo chứng minh trên 2r  1 có ước nguyên tố s và s > r
Mà 2r  1/ 2n  1  s / 2n  1  s / n , vô lý, vậy nếu n thỏa mãn ycbt thì n không có
ước nguyên tố khác 3. Vậy n  3k
Nếu k =1  23  1  9  3,9 chỉ có 1 ước nguyên tố là 3  n  3 thỏa mãn.
Nếu k  2 thì 19 / 29  1 mà 29  1/ 2n  1  k  2 loại
Vấy nếu n = 3 là số tự nhiên duy nhất thỏa mãn ycbt.


Bài 17 (Brasil-2009): Cho p, q  P, q  2 p  1 . Chứng minh rằng tồn tại một bội
của q mà tổng các chữ số của nó nhỏ hơn 4.
Lời giải
Nếu q = 5 thì n  10 5 và tổng các chữ số là 1  q  5 (đúng)
Nếu (q,5)=1, thì (q,10)=1. Theo định lý Fermat: 102 p  1(mod q)
 (10 p  1)(10 p  1)  0(mod q)

Nếu 10 p  1  0(mod q ) thì 10 p  1 là bội của q mà tổng các chữ số bằng 2.
Nếu 10 p  1(mod q ) , gọi h  ord p (10)  h / p  h 1, p

Nếu h  1  9  0(mod q )  q  3 vô lý
Nếu h = p .Đặt 10i  ri (mod q ),  1  ri  2 p , i  1, 2,..., p
.Vì h = p nên ri  rj (i, j ) :1  i  j  p Bây giờ ta chia tập 1, 2,...., 2p thành n tập j
con rời nhau 1, 2 p  1 ,2, 2 p  2 ,...., p  1, p  1 , p, 2 p ,
Nếu i : ri  p chọn n  2.10i  1  n  2 p  1(mod q)  0(mod q), S (n)  3
Nếu i : ri  2 p chọn n  10i  1  n  2 p  1(mod q )  0(mod q ),S(n)  2
Nếu ri  p, 2 p i {1, 2,..., p}thì theo nguyên tắc Diricle thì tồn tại i, j : ri  rj  2 p
Trong trường hợp này ta chọn
n  10i  10 j  1  n  2 p  1(mod q )  0(mod q ), S (n )  3

Bài toán được chứng minh hoàn toàn
Bài 18 : Tìm tất cả các số nguyên tố p,q sao cho a  N *.a3 pq  a(mod 3 pq)
Lời giải
Không giảm tổng quát ta giả sử p  q chọn
a  2  8 pq  2(mod 3 pq )  p  q  1(mod 2)

Theo định lý Ferma a3 pq  a3 p (mod q)a  N *
Theo giả thiết
a3 pq  a(mod 3 pq)  a3 pq  a(mod q )  a3 p  a(mod q )  a3 p1  1(mod q )a  N * , (a , q )  1

Gọi a là căn nguyên thủy của q thì ord(a)= q – 1 do q – 1/3p – 1 ta có
4q  4  4 p  4 và 3p – 1 không là bội của 3  3 p  1{q  1, 2(q  1)}


Nếu 3 p  1  q  1  q  3 p vô lý vì q nguyên tố
Nếu 3 p  1  2(q  1)  q 

3 p 1
2


Chứng minh tương tự p  1/ 3q  1  p  1/ 9 p 1  p 1/10  p  3, p  11
Nếu p  3  q  5 cặp (3,5) không thỏa mãn.
Nếu p  11  q  17 thì dễ thấy cặp (11,17) thỏa mãn.
Vậy (11,17) , (17,11) là hai cặp thỏa mãn yêu cầu bài toán.
Bài 19 : (Serbia-MO-2010) Cho một bảng n x n ô vuông được điền bởi các số
1,2,…, n 2 . Ta gọi n ô vông của bảng là rải rác nếu n ô này không có hai ô nào
cùng hàng hoặc cùng cột. Một số n được gọi là số đẹp nếu bảng n x n có thể
điền sao cho tích các số trên n ô rải rác bất kỳ có cùng số dư khi chia hết cho
n 2  1 . Hỏi n sau có đẹp hay không nếu :
a) n = 8
b) n = 10
Lời giải
a) Nếu n = 8 là số đẹp. Xét bảng 8 x 8 thỏa mãn ycbt. Khi đó tích của 8 ô rải
rác bất kỳ đều đồng dư với nhau theo mod 65 và đồng dư với S nghĩa là
64!  S 8 (mod 65) mà 64!  0(mod 65)  S 8  0(mod 65)  S  0(mod 65) mà 65 =
5.13 vậy trong 8 bộ này phải có ít nhất 8 số chia hết cho 13. Vậy 8 không
là số đẹp.
b) n  10  102  1  101 P a là căn nguyên thủy của 101. Ta coi 10 số điền
vào bảng là a1 , a2 ,...,a n

2

Điền vào ô(i,j) số 1010(i 1) j  tích các số trên n ô rời rạc là :
a10(1 2... q ) 1 2...10  a505  n  10 là số đẹp

Bài 20 : ( Mogolia 2014) Cho p là số nguyên tố lẻ {a1 , a2 ,..., a p 1}  {1, 2,..., p  1}
chứng minh rằng (b1 , b2 ,..., bp 1 ) là hoán vị của {a1 , a2 ,..., a p 1} sao cho
p 1

a


bi
i

 0(mod p )

i

Lời giải


p  P . Gọi g là căn nguyên thủy của p ta có {g1 , g 2 ,..., g p 1} là hệ thặng dư thu

gọn theo modp bài toán trở thành chứng minh {k1 , k2 ,..., k p 1} là hoán vị của
{1, 2,..., p  1} sao cho

p 1

g

ik j

 0(mod p )

i


 ki 
Ta chọn 
k 

 i

p 1
p 1
p 1
 i, i  {1,...,
}
p 1
p 1
p 1
2
i(
i )
ik
2
2
 S   g j  g 2   g i ( p i )
p 1
p 1
p 1
i 1
i 1
i
 p  i, i  {
,..., p}
2
2
2

p 1

p ( p 1)
p2  p (
)
p 1
p 1
p 1
ij
( p i )(p  j)
ij
ij
2


]
 (g  g
 g [1  g
 g (1  g 2 )
2 i, j
2 i  j  p 1
2 i  j  p 1
2

Ta có g

p ( p 1)
2

 (g

 ( p)

2

2

) p  1(mod p)  S  0(mod p) (đpcm)

Bài 21: (IMO-Shortlist-2011) p,q  Zt [ x] và p(n), q(n)  N* n  N* biết rằng
n  N * (P(n), q(n))  k, k  N* cho trước và 2q( n )  1/ 3 p ( n )  1. Chứng minh rằng
q( x)  C x  R

Lời giải
Với mỗi n nguyên dương ta đặt tương ứng M n  2q ( n )  1 . Gọi a,b là cấp của 2,3
theo mod M n dễ thấy a  q (n), b / p(n) . Do vậy từ giả thiết ta suy ra
gcd(a, b)  d .Vì diều này đúng với mọi n nên ta luôn tìm được x,y nguyên dương
1  n  ax  by  d ta thấy a / q(n  ax)  q(n) nên M (n)  2q ( n )  1/ 2q ( n )ax  1 lại có
2q ( n  ax )  1/ 3 p ( n  ax )  1 và ta cũng thấy rằng b / p(n  ax)  p( n  ax  by) nên ta suy ra
2q ( n )  1/ 3 p ( n  a x by )  1 . Như vậy với n nguyên dương bấy kỳ thì 2q ( n )  1 luôn là
ước của 3 p (i )  1 (i  d ) nào đó. Do vậy 2q ( n )  1  A với A là số lớn nhất trong các
số 3 p (i )  1 (i  d )
Điều này chỉ có thể xảy ra khi Q(n) là hằng số với mọi n nguyên dương suy ra
Q(x) phải là đa thức hằng. Ta có điều phải chứng minh
Bài 22 : (IMO-Shortlist-2012) Cho x,y là hai số nguyên dương thỏa mãn điều
kiện x 2  1 là bội của 2n y  1 với mọi n nguyên dương. Chứng minh rằng x = 1
n

Lời giải
Giả sử tồn tại x,y thỏa mãn yêu cầu bài toán. Khi đó xét p là một ước nguyên tố
của 2n y  1 thì p / x2 1 . Gọi h là cấp của x mod p thì ta phải có h / gcd(2n , p 1)
như vậy nếu p có dạng 4k + 3 thì h / 2 suy ra p / x 2  1
n



Ta thấy rằng nếu x > 1 thì số lượng nguyên tố p là ước của x 2  1 là hữu hạn.
Như vậy để chứng minh x = 1 ta sẽ chỉ ra rằng tập các ước nguyên tố dạng
4k + 3 của 2n y  1 là vô hạn.
Rõ ràng ta chỉ cần xét với y lẻ. Đặt 2 y  1  p1e p2e ... pre ta giả sử phản chứng rằng
ngoài r số kể trên chỉ có hữu hạn số nguyên tố dạng 4k + 3 là pr 1 , pr 2 ,..., pr t mà
có dạng 2n y  1
1

2

r

Để có được thêm một ước nguyên tố dạng 4k + 3 ta sẽ đi tìm một số n mà
2n y  1
2n y  1
là số chia 4 dư 3 nên có
2 y  1/ 2 y  1 và gcd(
, 2 y  1)  1 khi đó A 
2 y 1
2 y 1
n

ước nguyên tố 4k +3. Ta cũng có thể chọn n thỏa mãn 2n y  1 không là bội của
các số pr 1 , pr 2 ,..., pr t để có q là một số nguyên tố dạng 4k +3 khác.
Để có những điều này ta chọn n  1   ( p1e 1 p2e  2 ... pre  r pr 1 pr  2 ... pr t )
i

i


r

khi đó 2n y  2 y  1(mod p1e 1 p2e  2 ... pre  r pr 1 pr  2 )
i

i

r

Ta được n cần chọn từ đó suy ra điều giả sử là sai. Vậy tập ước nguyên tố dạng
4k +3 của 2n y  1 là vô hạn. Từ đây ta có điều phải chứng minh.

Lời kết
Bài viết trên là việc tổng kết bài giảng chuyên đề số học cho đội tuyển Hà Nội
từ năm 2012 tới nay. Sau bài viết này tôi rất mong được sự quan tâm trao đổi
của các bạn đồng nghiệp để tôi có thể hoàn thiện bài giảng tốt hơn trong những
năm tới.



×